Difference between revisions of "2005 USAMO Problems/Problem 6"
(→See Also) |
5849206328x (talk | contribs) |
||
Line 1: | Line 1: | ||
− | ==Problem== | + | == Problem == |
− | For <math>m</math> a positive integer, let <math>s(m)</math> be the sum of the digits of <math>m</math>. For <math>n\ge 2</math>, let <math>f(n)</math> be the minimal <math>k</math> for which there exists a set <math>S</math> of <math>n</math> positive integers such that <math>s\left(\sum_{x\in X} x\right) = k</math> for any nonempty subset <math>X\subset S</math>. Prove that there are constants <math>0 < C_1 < C_2</math> with | + | (''Titu Andreescu, Gabriel Dospinescu'') For <math>m</math> a positive integer, let <math>s(m)</math> be the sum of the digits of <math>m</math>. For <math>n\ge 2</math>, let <math>f(n)</math> be the minimal <math>k</math> for which there exists a set <math>S</math> of <math>n</math> positive integers such that <math>s\left(\sum_{x\in X} x\right) = k</math> for any nonempty subset <math>X\subset S</math>. Prove that there are constants <math>0 < C_1 < C_2</math> with |
− | <cmath> | + | <cmath>C_1 \log_{10} n \le f(n) \le C_2 \log_{10} n.</cmath> |
− | C_1 \log_{10} n \le f(n) \le C_2 \log_{10} n. | ||
− | </cmath> | ||
− | ==Solution== | + | == Solution == |
− | {{ | + | For the upper bound, let <math>p</math> be the smallest integer such that <math>10^p\geq n(n+1)/2</math> and let |
− | + | <cmath>S = \{10^p - 1, 2(10^p - 1), \ldots, n(10^p - 1)}.</cmath> | |
+ | The sum of any nonempty set of elements of <math>S</math> will have the form <math>k(10^p - 1)</math> for some <math>1\leq k\leq n(n+1)/2</math>. Write <math>k(10^p - 1) = [(k - 1)10^p] + [(10^p - 1) - (k - 1)]</math>. The second term gives the bottom <math>p</math> digits of the sum and the first term gives at most <math>p</math> top digits. Since the sum of a digit of the second term and the corresponding digit of <math>k - 1</math> is always 9, the sum of the digits will be <math>9p</math>. Since <math>10^{p-1} < n(n+1)/2</math>, this example shows that | ||
+ | <cmath>f(n)\leq 9p\leq 9\log_{10}(5n(n+1)).</cmath> | ||
+ | Since <math>n\geq 2</math>, <math>5(n + 1) < n^4</math>, and hence | ||
+ | <cmath>f(n) < 9\log_{10}n^5 = 45\log_{10}n.</cmath> | ||
+ | For the lower bound, let <math>S</math> be a set of <math>n\geq 2</math> positive integers such that any nonempty <math>X\subset S</math> has <math>s\left(\sum_{x\in X} x\right) = f(n)</math>. Since <math>s(m)</math> is always congruent to <math>m</math> modulo 9, <math>\sum_{x\in X} x\equiv f(n)\pmod{9}</math> for all nonempty <math>X\subset S</math>. Hence every element of <math>S</math> must be a multiple of 9 and <math>f(n)\geq 9</math>. Let <math>q</math> be the largest positive integer such that <math>10^q - 1\leq n</math>. Lemma 1 below shows that there is a nonempty subset <math>X</math> of <math>S</math> with <math>\sum_{x\in X} x</math> a multiple of <math>10^q - 1</math>, and hence Lemma 2 shows that <math>f(n)\geq 9q</math>. | ||
− | == See Also== | + | '''Lemma 1.''' Any set of <math>m</math> positive integers contains a nonempty subset whose sum is a multiple of <math>m</math>. |
+ | |||
+ | ''Proof.'' Suppose a set <math>T</math> has no nonempty subset with sum divisible by <math>m</math>. Look at the possible sums mod <math>m</math> of nonempty subsets of <math>T</math>. Adding a new element <math>a</math> to <math>T</math> will give at least one new sum mod <math>m</math>, namely the least multiple of <math>a</math> which does not already occur. Therefore the set <math>T</math> has at least <math>|T|</math> distinct sums mod <math>m</math> of nonempty subsets and <math>|T| < m</math>. <math>\blacksquare</math> | ||
+ | |||
+ | '''Lemma 2.''' Any positive multiple <math>M</math> of <math>10^q - 1</math> has <math>s(M)\geq 9q</math>. | ||
+ | |||
+ | ''Proof.'' Suppose on the contrary that <math>M</math> is the smallest positive multiple of <math>10^q - 1</math> with <math>s(M) < 9q</math>. Then <math>M\neq 10^q - 1</math>, hence <math>M > 10^q</math>. Suppose the most significant digit of <math>M</math> is the <math>10^m</math> digit, <math>m\geq q</math>. Then <math>N = M - 10^{m-q}(10^q - 1)</math> is a smaller positive multiple of <math>10^q - 1</math> and has <math>s(N)\leq s(M) < 9q</math>, a contradiction. <math>\blacksquare</math> | ||
+ | |||
+ | Finally, since <math>10^{q+1} > n</math>, we have <math>q + 1 > \log_{10} n</math>. Since <math>f(n)\geq 9q</math> and <math>f(n)\geq 9</math>, we have | ||
+ | <cmath>f(n)\geq \frac{9q + 9}{2} > \frac{9}{2}\log_{10}n.</cmath> | ||
+ | Weaker versions of Lemmas 1 and 2 are still sufficient to prove the desired type of lower bound. | ||
+ | |||
+ | {{alternate solutions}} | ||
+ | |||
+ | == See Also == | ||
{{USAMO newbox|year=2005|num-b=5|after=Last Question}} | {{USAMO newbox|year=2005|num-b=5|after=Last Question}} | ||
{{MAA Notice}} | {{MAA Notice}} | ||
− |
Revision as of 20:43, 27 July 2014
Problem
(Titu Andreescu, Gabriel Dospinescu) For a positive integer, let be the sum of the digits of . For , let be the minimal for which there exists a set of positive integers such that for any nonempty subset . Prove that there are constants with
Solution
For the upper bound, let be the smallest integer such that and let
\[S = \{10^p - 1, 2(10^p - 1), \ldots, n(10^p - 1)}.\] (Error compiling LaTeX. Unknown error_msg)
The sum of any nonempty set of elements of will have the form for some . Write . The second term gives the bottom digits of the sum and the first term gives at most top digits. Since the sum of a digit of the second term and the corresponding digit of is always 9, the sum of the digits will be . Since , this example shows that Since , , and hence For the lower bound, let be a set of positive integers such that any nonempty has . Since is always congruent to modulo 9, for all nonempty . Hence every element of must be a multiple of 9 and . Let be the largest positive integer such that . Lemma 1 below shows that there is a nonempty subset of with a multiple of , and hence Lemma 2 shows that .
Lemma 1. Any set of positive integers contains a nonempty subset whose sum is a multiple of .
Proof. Suppose a set has no nonempty subset with sum divisible by . Look at the possible sums mod of nonempty subsets of . Adding a new element to will give at least one new sum mod , namely the least multiple of which does not already occur. Therefore the set has at least distinct sums mod of nonempty subsets and .
Lemma 2. Any positive multiple of has .
Proof. Suppose on the contrary that is the smallest positive multiple of with . Then , hence . Suppose the most significant digit of is the digit, . Then is a smaller positive multiple of and has , a contradiction.
Finally, since , we have . Since and , we have Weaker versions of Lemmas 1 and 2 are still sufficient to prove the desired type of lower bound.
Alternate solutions are always welcome. If you have a different, elegant solution to this problem, please add it to this page.
See Also
2005 USAMO (Problems • Resources) | ||
Preceded by Problem 5 |
Followed by Last Question | |
1 • 2 • 3 • 4 • 5 • 6 | ||
All USAMO Problems and Solutions |
The problems on this page are copyrighted by the Mathematical Association of America's American Mathematics Competitions.